Difference between revisions of "2019 AMC 10A Problems/Problem 4"

(Solution)
(See Also)
Line 7: Line 7:
 
<math>\textbf{(A) } 75 \qquad\textbf{(B) } 76 \qquad\textbf{(C) } 79 \qquad\textbf{(D) } 84 \qquad\textbf{(E) } 91</math>
 
<math>\textbf{(A) } 75 \qquad\textbf{(B) } 76 \qquad\textbf{(C) } 79 \qquad\textbf{(D) } 84 \qquad\textbf{(E) } 91</math>
  
==See Also==
+
==Solution==
 
 
{{AMC10 box|year=2019|ab=A|num-b=3|num-a=5}}
 
{{AMC12 box|year=2019|ab=A|num-b=2|num-a=4}}
 
{{MAA Notice}}
 

Revision as of 18:03, 9 February 2019

The following problem is from both the 2019 AMC 10A #4 and 2019 AMC 12A #3, so both problems redirect to this page.

Problem

A box contains $28$ red balls, $20$ green balls, $19$ yellow balls, $13$ blue balls, $11$ white balls, and $9$ black balls. What is the minimum number of balls that must be drawn from the box without replacement to guarantee that at least $15$ balls of a single color will be drawn$?$

$\textbf{(A) } 75 \qquad\textbf{(B) } 76 \qquad\textbf{(C) } 79 \qquad\textbf{(D) } 84 \qquad\textbf{(E) } 91$

Solution